At which point along the x axis will #q_2# experience no net force given #d=2 m# and relation between charges 1 and 3 is #3q_1=q_3#?

1 Answer
Mar 10, 2018

Let charge #q_1# be placed at origin. #:.q_3# is located at #2hati#.
Let #q_2# be at #xhati# to meet the required condition.

Force on #q_2# due to #q_1# using Coulombs' Law

#vecF_(q_1to q_2)=k_e(q_1q_2)/x^2hati#

Similarly force on #q_2# due to #q_3#

#vecF_(q_3to q_2)=-k_e(q_3q_2)/(d-x)^2hati#

For net force on #q_2=0#, sum of above two forces must be zero. We have

#vecF_(q_1to q_2)+vecF_(q_3to q_2)=0=k_e(q_1q_2)/x^2hati+(-k_e(q_3q_2)/(d-x)^2hati)#
#=>(q_1)/x^2=(q_3)/(d-x)^2#

Inserting value of charges #3q_1=q_3#

#(q_1)/x^2=(3q_1)/(d-x)^2#
#=>1/x^2=(3)/(d-x)^2#
#=>3x^2-(d-x)^2=0#
#=>(sqrt3x)^2-(d-x)^2=0#
#=>[sqrt3x+(d-x)][sqrt3x-(d-x)]=0#
#=>[d+x(sqrt3-1)][x(sqrt3+1)-d]=0#

Two roots are

#[d+x(sqrt3-1)]=0and [x(sqrt3+1)-d]=0#
#x=-d/(sqrt3-1)and x=d/(sqrt3+1)#

Rationalizing denominators we get

#x=-d/2(sqrt3+1)and x=d/2(sqrt3-1)#

Inserting given value of #d# we get

#x=-(sqrt3+1)\ m#
# x=(sqrt3-1)\ m#

Both solutions are valid if we consider only the magnitude of the net force. When we consider direction as well we find that only valid solution is when #q_2# is placed between the two charges #q_1and q_3#.

Hence, #x=(sqrt3-1)\ m#